Difference between revisions of "1997 JBMO Problems/Problem 4"

(WIP Solution)
m (Conciseness change (thanks Quantomaticguy))
 
(3 intermediate revisions by 2 users not shown)
Line 3: Line 3:
 
Determine the triangle with sides <math>a,b,c</math> and circumradius <math>R</math> for which <math>R(b+c) = a\sqrt{bc}</math>.
 
Determine the triangle with sides <math>a,b,c</math> and circumradius <math>R</math> for which <math>R(b+c) = a\sqrt{bc}</math>.
  
== Solution ==
+
== Solutions ==
 +
 
 +
===Solution 1===
  
 
Solving for <math>R</math> yields <math>R = \tfrac{a\sqrt{bc}}{b+c}</math>.  We can substitute <math>R</math> into the area formula <math>A = \tfrac{abc}{4R}</math> to get
 
Solving for <math>R</math> yields <math>R = \tfrac{a\sqrt{bc}}{b+c}</math>.  We can substitute <math>R</math> into the area formula <math>A = \tfrac{abc}{4R}</math> to get
Line 11: Line 13:
 
&= \frac{(b+c)\sqrt{bc}}{4}.
 
&= \frac{(b+c)\sqrt{bc}}{4}.
 
\end{align*}</cmath>
 
\end{align*}</cmath>
We also know that <math>A = \tfrac{1}{2}ab \sin(\theta)</math>, where <math>\theta</math> is the angle between sides <math>b</math> and <math>c.</math>  Substituting this yields
+
We also know that <math>A = \tfrac{1}{2}bc \sin(\theta)</math>, where <math>\theta</math> is the angle between sides <math>b</math> and <math>c.</math>  Substituting this yields
 
<cmath>\begin{align*}
 
<cmath>\begin{align*}
\tfrac{1}{2}ab \sin(\theta) &= \frac{(b+c)\sqrt{bc}}{4} \\
+
\tfrac{1}{2}bc \sin(\theta) &= \frac{(b+c)\sqrt{bc}}{4} \\
 
2\sqrt{bc} \cdot \sin(\theta) &= b+c \\
 
2\sqrt{bc} \cdot \sin(\theta) &= b+c \\
 
\sin(\theta) &= \frac{b+c}{2\sqrt{bc}}
 
\sin(\theta) &= \frac{b+c}{2\sqrt{bc}}
 
\end{align*}</cmath>
 
\end{align*}</cmath>
Since <math>\theta</math> is inside a triangle, <math>0 < \sin{\theta} \le 1</math>.
+
Since <math>\theta</math> is inside a triangle, <math>0 < \sin{\theta} \le 1</math>.  Substitution yields
 +
<cmath>0 < \frac{b+c}{2\sqrt{bc}} \le 1.</cmath>
 +
Note that <math>2\sqrt{bc}</math>, so multiplying both sides by that value would not change the inequality sign.  This means
 +
<cmath>0 < b+c \le 2\sqrt{bc}.</cmath>
 +
However, by the [[AM-GM Inequality]], <math>b+c \ge 2\sqrt{bc}</math>.  Thus, the equality case must hold, so <math>b = c</math> where <math>b, c > 0</math>.  When plugging <math>b = c</math>, the inequality holds, so the value <math>b=c</math> truly satisfies all conditions.
  
 +
<br>
 +
That means <math>\sin(\theta) = \frac{2b}{2\sqrt{b^2}} = 1,</math> so <math>\theta = 90^\circ.</math>  That means the only truangle that satisfies all the conditions is a 45-45-90 triangle where <math>a</math> is the longest side.  In other words, <math>(a,b,c) \rightarrow \boxed{(n\sqrt{2},n,n)}</math> for all positive <math>n.</math>
  
 
== See Also ==
 
== See Also ==

Latest revision as of 15:48, 23 February 2021

Problem

Determine the triangle with sides $a,b,c$ and circumradius $R$ for which $R(b+c) = a\sqrt{bc}$.

Solutions

Solution 1

Solving for $R$ yields $R = \tfrac{a\sqrt{bc}}{b+c}$. We can substitute $R$ into the area formula $A = \tfrac{abc}{4R}$ to get \begin{align*} A &= \frac{abc}{4 \cdot \tfrac{a\sqrt{bc}}{b+c} } \\ &= \frac{abc}{4a\sqrt{bc}} \cdot (b+c) \\ &= \frac{(b+c)\sqrt{bc}}{4}. \end{align*} We also know that $A = \tfrac{1}{2}bc \sin(\theta)$, where $\theta$ is the angle between sides $b$ and $c.$ Substituting this yields \begin{align*} \tfrac{1}{2}bc \sin(\theta) &= \frac{(b+c)\sqrt{bc}}{4} \\ 2\sqrt{bc} \cdot \sin(\theta) &= b+c \\ \sin(\theta) &= \frac{b+c}{2\sqrt{bc}} \end{align*} Since $\theta$ is inside a triangle, $0 < \sin{\theta} \le 1$. Substitution yields \[0 < \frac{b+c}{2\sqrt{bc}} \le 1.\] Note that $2\sqrt{bc}$, so multiplying both sides by that value would not change the inequality sign. This means \[0 < b+c \le 2\sqrt{bc}.\] However, by the AM-GM Inequality, $b+c \ge 2\sqrt{bc}$. Thus, the equality case must hold, so $b = c$ where $b, c > 0$. When plugging $b = c$, the inequality holds, so the value $b=c$ truly satisfies all conditions.


That means $\sin(\theta) = \frac{2b}{2\sqrt{b^2}} = 1,$ so $\theta = 90^\circ.$ That means the only truangle that satisfies all the conditions is a 45-45-90 triangle where $a$ is the longest side. In other words, $(a,b,c) \rightarrow \boxed{(n\sqrt{2},n,n)}$ for all positive $n.$

See Also

1997 JBMO (ProblemsResources)
Preceded by
Problem 3
Followed by
Problem 5
1 2 3 4 5
All JBMO Problems and Solutions